Đến nội dung

Stranger411 nội dung

Có 85 mục bởi Stranger411 (Tìm giới hạn từ 07-06-2020)



Sắp theo                Sắp xếp  

#343069 bài toán về tập tốt

Đã gửi bởi Stranger411 on 03-08-2012 - 12:53 trong Tổ hợp và rời rạc

Bài toán đã có ở đây :D

Bulgaria TST 2003



#312379 $\frac{1}{2-\cos A}+\frac{1}{2-\cos B}+\frac{1}...

Đã gửi bởi Stranger411 on 24-04-2012 - 10:28 trong Bất đẳng thức - Cực trị

Bài 1: Cho $\Delta ABC$. Chứng minh:
$$\frac{1}{2-\cos A}+\frac{1}{2-\cos B}+\frac{1}{2-\cos C}\ge 2$$


Bài 2: Cho $\Delta ABC$. Chứng minh:
$$\frac{\left( 1-\sin \frac{A}{2} \right)\left( 1+\cos \frac{A}{2} \right)}{\sin \frac{A}{2}\left( 1+\sin \frac{A}{2} \right)}+\frac{\left( 1-\sin \frac{B}{2} \right)\left( 1+\cos \frac{B}{2} \right)}{\sin \frac{B}{2}\left( 1+\sin \frac{B}{2} \right)}+\frac{\left( 1-\sin \frac{C}{2} \right)\left( 1+\cos \frac{C}{2} \right)}{\sin \frac{C}{2}\left( 1+\sin \frac{C}{2} \right)}\ge 2+\sqrt{3}$$



#312728 Cho $a,b,c>0$và $ab+bc+ca+abc=4$ .CM$a^3+b^3+c^3...

Đã gửi bởi Stranger411 on 25-04-2012 - 23:19 trong Bất đẳng thức và cực trị

Cho$a,b,c>0$ thỏa mãn $ab+bc+ca+abc=4$.Chứng minh
$a^3+b^3+c^3+9abc\geq 4(a+b+c)$

Phải nói đây một bất đẳng thức khá bá đạo.
Nhưng nếu làm bđt nhiều thì cũng không khó gì để nhận ra nó.
Đổi biến $a=\frac{2x}{y+z},b=\frac{2y}{x+z},c=\frac{2z}{x+y} $, ta được $ab+bc+ca+abc=4$
Lời giải còn lại của bài toán các bạn xem trong file đính kèm.

File gửi kèm




#312448 $\frac{1}{2-\cos A}+\frac{1}{2-\cos B}+\frac{1}...

Đã gửi bởi Stranger411 on 24-04-2012 - 19:17 trong Bất đẳng thức - Cực trị

Hình đã gửiÔng bạn của mình đây mà
Trước hết ta sẽ chứng minh $$cosA+cosB+cosC\geq -\frac{3}{2}$$
Cho $\overrightarrow{OA};\overrightarrow{OB};\overrightarrow{OC}$ là 3 véc tơ đơn vị. ( các góc $A,B,C \in (0; \pi)$)
Ta dễ dàng chứng minh được $(\overrightarrow{OA}+\overrightarrow{OB}+\overrightarrow{OC})^2=3+2(cosA+cosB+cosC) \ge 0$
Từ đây suy ra $$cosA+cosB+cosC\geq \frac{-3}{2}$$
Áp dụng bất đẳng thức AM-HM ta có:
$$\frac{1}{2-cosA}+\frac{1}{2-cosB}+\frac{1}{2-cosC}\geq \frac{9}{3-cosA-cosB-cosC}\geq \frac{9}{3+\frac{3}{2}}=2$$

Một lỗi sai khá cơ bản đó ông bạn Hình đã gửi
$$2\cos (\overrightarrow{OA},\overrightarrow{OB})= \cos2C$$
Từ đó, ta có:
$$cos2A+cos2B+cos2C\geq \frac{-3}{2}$$
Bài này không đơn giản như vậy đâu



#359606 Tìm nghiệm nguyên

Đã gửi bởi Stranger411 on 06-10-2012 - 22:43 trong Số học

Tìm $p,q\in \mathbb{P}$ thỏa mãn $3pq\mid a^{3pq}-a$ với mọi $a\in \mathbb{Z}^+$

Nếu biết giới hạn $p,q$ và chọn $a$ là căn nguyên thủy của $n$ ngay từ đầu thì bài toán sẽ gọn hơn rất nhiều.
Giả sử $p \ge q$

+ Cho $a=3$, ta có:
${3^{pq}} \equiv 3\left( {\bmod 3pq} \right) \Rightarrow 3\left( {{3^{pq - 1}} - 1} \right) \vdots 3pq \Rightarrow p,q > 3$

+ Cho ${a^{\varphi \left( n \right)}} \equiv 1\left( {\bmod n} \right)$ ( $a$ là căn nguyên thủy của $n$)
Theo định lí Fermat nhỏ: ${a^{p - 1}} \equiv 1\left( {\bmod p} \right)$
Vì ${a^{3pq - 1}} \equiv 1\left( {\bmod p} \right) \Rightarrow p - 1|3pq - 1 \Rightarrow p - 1|3q - 1$
Chứng minh tương tự: $q - 1|3p - 1$
Vì $p \ge q$ nên $3q - 1 \in \left\{ {p - 1;2\left( {p - 1} \right);3\left( {p - 1} \right)} \right\}$
Thay vào điều kiện bài toán, ta được: $\boxed{(p,q)=(11,17),(17,11)}$



#359617 $\frac{{d({n^2})}}{{d(n)...

Đã gửi bởi Stranger411 on 06-10-2012 - 23:04 trong Số học

Với mỗi số nguyên dương n,kí hiệu d(n) là số các ước dương của n.Tìm tất cả số nguyên dương m sao cho tồn tại số nguyên dương thỏa $\frac{{d({n^2})}}{{d(n)}} = m$

Đây là bài 3 của IMO 1998
Các bạn có thể tham khảo một số lời giải khác ở đây:
http://www.artofprob...=124439#p124439



#345818 $\frac{{{a^2} - 2}}{{2...

Đã gửi bởi Stranger411 on 11-08-2012 - 12:21 trong Số học

Tìm $a,b \in {\mathbb{Z}^ + }$ sao cho $\frac{{{a^2} - 2}}{{2{b^2} + 3}} \in \mathbb{Z}$



#340870 Chứng minh: $a = {10^k}$

Đã gửi bởi Stranger411 on 27-07-2012 - 20:43 trong Số học

Cho n=1.Ta có S(a+1)=2
Suy ra a+1 có dạng:\[a + 1 = 2 \times {10^k}\] hoặc \[a + 1 = {10^k} + {10^h}(k > h)\]

Đến đây có thể dùng 2 tính chất quan trọng của $S(n)$ để giải bài toán.
là $S(m)+S(n) \ge S(m+n)$ và $S(m)S(n) \ge S(mn)$



#340310 Chứng minh: $a = {10^k}$

Đã gửi bởi Stranger411 on 26-07-2012 - 01:01 trong Số học

Cho số nguyên dương $a$ thỏa mãn $S\left( {{a^n} + n} \right) = 1 + S\left( n \right)$ với mọi số tự nhiên $n$ lớn tùy ý.
Chứng minh $a$ là một lũy thừa của $10$.

- Kvant -




#409245 Tìm số dư của phép chia $S=\prod^{p}_{t=1}(t^...

Đã gửi bởi Stranger411 on 30-03-2013 - 22:03 trong Số học

Bài toán:

Ch0 số nguyên tố lẻ $p=mk+2$ tr0ng đó $m.k\in \mathbb{N}^{*},m>2$. Tìm số dư của phép chia $S=\prod^{p}_{t=1}(t^{m-1}+t^{m-2}+...+t+1)$ ch0 $p$.

Một bài khó đánh giá hơn ;)

Cho số nguyên tố $p \equiv 1( \bmod m)$, $m  >2$. Chứng minh:
\[\prod\limits_{t = 1}^p {\left( {{t^{m - 1}} + {t^{m - 2}} +  \ldots  + 1} \right)}  \equiv 0(\bmod p)\]




#353030 $p \equiv 1 (\bmod m)$

Đã gửi bởi Stranger411 on 08-09-2012 - 23:40 trong Số học

Tổng quát hóa từ một bài toán:
Cho số nguyên tố $p \equiv 1 (\bmod m)$ với $m>2$. Chứng minh:
\[\prod\limits_{i = 1}^p {\left( {{i^{m - 1}} + {i^{m - 2}} + \ldots + i + 1} \right)} \equiv 0 (\bmod p)\]


Bài này dùng 1 tí kiến thức về hệ thu gọn.
Bên mathlink post mấy tháng rồi mà chưa có lời giải :-<



#423590 Chứng minh 3 điểm cùng thuộc một đường vuông góc với $OE'$

Đã gửi bởi Stranger411 on 03-06-2013 - 23:15 trong Hình học

Cho tam giác $ABC$ có $O$ là tâm đường tròn ngoại tiếp, $E$ là tâm đường tròn Euler. Lấy $E'$ thỏa $\widehat{E'BA}=\widehat{EBC}$ và $\widehat{E'AB}=\widehat{EAC}$. Trung trực $OA$ cắt $BC$ tại $A'$. Các điểm $B',C'$ được xác định tương tự. Chứng minh $A',B',C'$ cùng thuộc 1 đường thẳng vuông góc với $OE'$

 

 

 

 




#314274 $$\dfrac{a^3+b^3+c^3+3abc}{(a+b+c)(ab+bc+ca)}+\dfrac{abc}...

Đã gửi bởi Stranger411 on 04-05-2012 - 13:03 trong Bất đẳng thức - Cực trị

Bài toán :
Cho $a, b, c \ge 0$ . Chứng minh rằng :
$$\dfrac{a^3+b^3+c^3+3abc}{(a+b+c)(ab+bc+ca)}+\dfrac{abc}{a^2b+b^2c+c^2a}\ge 1$$
Nguồn : ML

Bài toán của bạn được ghép từ 3 bổ đề sau:
1)${{a}^{3}}+{{b}^{3}}+{{c}^{3}}+3abc\ge \sum{ab\left( a+b \right)}$


2)$\left( a+b+c \right)\left( ab+bc+ca \right)\frac{8}{9}\le \left( a+b \right)\left( b+c \right)\left( c+a \right)$

3)$\frac{a}{b+c}+\frac{b}{c+a}+\frac{c}{a+b}+\frac{3abc}{2\left( {{a}^{2}}b+{{b}^{2}}c+{{c}^{2}}a \right)}\ge 2$



#312383 CMR: $xy+yz+zx\leq 8$

Đã gửi bởi Stranger411 on 24-04-2012 - 10:36 trong Bất đẳng thức và cực trị

Cho $x,y,z$ la nghiệm của hệ pt:$\left\{\begin{matrix} x^2+xy+y^2=3\\ y^2+yz+z^2=16\end{matrix}\right.$
CMR: $xy+yz+zx\leq 8$

Áp dụng bất đẳng thức Cauchy-Schwarz, ta có:
${{\left( xy+yz+zx \right)}^{2}}$
$=\left[ x\left( y+\frac{z}{2} \right)+z\left( y+\frac{x}{2} \right) \right]$
$\le \left[ {{x}^{2}}+\frac{4}{3}{{\left( y+\frac{x}{2} \right)}^{2}} \right]\left[ {{\left( y+\frac{z}{2} \right)}^{2}}+\frac{3}{4}{{z}^{2}} \right]$
$=\frac{4}{3}\left( {{x}^{2}}+xy+{{y}^{2}} \right)\left( {{y}^{2}}+yz+{{z}^{2}} \right)=64$
$\Leftrightarrow xy+yz+zx\le 8$ (đpcm)



#432488 Hỏi có thể thực hiện dc phép tô màu nói trên hay không nếu: 1.$n= 2012...

Đã gửi bởi Stranger411 on 03-07-2013 - 11:37 trong Tổ hợp và rời rạc

xét số nguyên $n> 1$. Người ta muốn tô màu tất cả các số tự nhiên bởi 2 màu xanh và đỏ sao cho các điều kiện sau được đồng thời thỏa mãn:

i. Mỗi số dc tô bởi 1 màu, và mỗi màu đều dc dùng để tô vô số số;

ii. tổng của n số đôi 1 khác nhau cùng màu là số có cùng màu đó.

Hỏi có thể thực hiện dc phép tô màu nói trên hay không nếu:

1.$n= 2012$

2.$n= 2013$

Tổng quát:
* Nếu $n$ lẻ thì phép tô thực hiện được:
Tô tất cả số chẵn cùng màu đỏ, số lẻ cùng màu xanh. Khi đó, mỗi số được tô bởi đúng 1 màu và có vô hạn lần tô mỗi màu (vì có vô hạn số lẻ, số chẵn). Tổng $n$ số cùng màu là một số cùng màu vì tổng $n$ số lẻ là một số lẻ (vì $n$ lẻ) và tổng $n$ số chẵn là một số chẵn.

* Nếu $n$ chẳn, thì phép tô không thực hiện được:
Phản chứng, giả sử phép tô thực hiện được, suy ra được có vô hạn số được tô bởi 2 màu xanh đỏ.
Khi đó, tồn tại số $a_1$ được tô màu xanh và $b_1 = a_1 +1$ được tô bỏi màu đỏ.

Tiếp tục, cũng tồn tại $b_2 > b_1$ mà $b_2$ được tổ bởi màu đỏ và $a_2$ được tổ bởi màu xanh.

Tương tư như vậy tồn tại $a_{n-1} > a_{n-2}$ được tô màu xanh và $b_{n-1} = a_{n-1} +1$ tô bởi màu đỏ và cũng tồn tại $b_n > b_{n-1}$ mà $b_n$ được tô màu đỏ và $a_n = b_n +1$ tô bởi màu xanh.

Tóm lại các số $a_i$ và $b_i$ là đôi một khác nhau và thỏa $a_i$ màu xanh và $b_i$ màu đỏ và $b_{2k-1} = a_{2k-1} +1$ và $b_{2k} = a_{2k} -1$ với mọi $k \le \frac{n}{2}$

Theo điều kiện đề bài thì gọi $a$ là tổng các số $a_i$ nên a được tô xanh còn $b$ là tổng các $b_i$ nên $b$ được tô đỏ. Nhưng vì $b_{2k-1} = a_{2k-1} +1$ và $b_{2k} = a_{2k} -1$ với mọi $k \le \frac{n}{2}$ nên suy ra $a=b$ từ đó $a,b$ phải được tô cùng màu dẫn tới vô lí.




#344948 CMR$\left( {\frac{{p - 1}}{2...

Đã gửi bởi Stranger411 on 08-08-2012 - 23:38 trong Số học

Bài toán trên còn 1 cách phát biểu khác như sau:
Hình đã gửi



#392819 Bất khả quy trên $\mathbb{Z[x]}$

Đã gửi bởi Stranger411 on 03-02-2013 - 15:29 trong Đa thức

Cho đa thức $P(x)= \prod_{i=1}^{n}( x- a_i)$ với $n \ge 5$ và các $a_i \in \mathbb{Z}$ phân biệt. Chứng minh nếu tam thức $ a x^2 + bx +1$ bất khả quy trên $\mathbb{Z[x]}$ thì đa thức $ a P(x)^2 + b P(x) +1$ cũng bất khả quy trên $\mathbb{Z[x]}$



#343066 Chứng minh giá trị của $k$ thuộc 1 tập xác định

Đã gửi bởi Stranger411 on 03-08-2012 - 12:48 trong Số học

Chứng minh rằng nếu tồn tại $ a;b\in\mathbb{Z}^{+} $ sao cho $ \frac{(a-b)^{2}+m}{pab-q}= k (m\in\mathbb{N}^{*}) $ với $p,q$ được xác định như trên thì giá trị của $k$ thuộc 1 tập xác định.



#392813 $a^2 +b | a + b^2$

Đã gửi bởi Stranger411 on 03-02-2013 - 14:57 trong Số học

Cho số nguyên tố $p$, tìm tất cả các cặp số nguyên dương $(a,b)$ thỏa 2 điều kiện:
1) $a^2 +b = p^k$ với $k \in \mathbb{Z^+}$
2) $a^2 +b | a + b^2$


ps: mấy chú ở Khtn chắc biết bài này nhỉ ;)



#346650 ${p_{k + 1}}|{p_1} + {p_2} +...

Đã gửi bởi Stranger411 on 14-08-2012 - 11:48 trong Số học

Cho số nguyên $n \ge 2$.
Chứng minh rằng tồn tại 1 bộ hoán vị ${p_1},{p_2}, \ldots ,{p_n}$ của $1,2, \ldots ,n$ sao cho ${p_{k + 1}}|{p_1} + {p_2} + \ldots + {p_k}$ với $k = 1,2, \ldots ,n-1$



#411157 $a^7 + 7 = b^2$

Đã gửi bởi Stranger411 on 07-04-2013 - 21:14 trong Số học

Tìm tất cả các cặp số nguyên dương $(a,b)$ thỏa mãn:
$$a^7+7=b^2$$




#347133 Tìm số nguyên $n> 1$ sao cho $\frac{2^{n...

Đã gửi bởi Stranger411 on 16-08-2012 - 10:06 trong Số học

Bài trên lâu lắm rồi :D Mình mở rộng bằng căn nguyên thủy tí nữa cho nó mạnh :D

Mở rộng: Tìm tất cả các số nguyên $n>1$ sao cho tồn tại duy nhất số nguyên $a$ với $0 < a < n!$ sao cho:
\[n!|{a^n} + 1\]

BÀI TOÁN: Xác định tất cả các số nguyên $n> 1$ sao cho $\frac{2^{n}+1}{n^{2}}$ là một số nguyên.

Bài này còn 2 cách giải nữa bằng căn nguyên thủy và LTE ;)



#347129 Tìm số nguyên $n> 1$ sao cho $\frac{2^{n...

Đã gửi bởi Stranger411 on 16-08-2012 - 09:52 trong Số học

BÀI TOÁN: Xác định tất cả các số nguyên $n> 1$ sao cho $\frac{2^{n}+1}{n^{2}}$ là một số nguyên.

Bài này ko cần phải dùng đến cấp của 1 số đâu :)

Bổ đề 1: Cho các số nguyên $m,n$ và $a>1$. Ta có: $\gcd \left( {{a^m} - 1,{a^n} - 1} \right) = {a^{\gcd \left( {m,n} \right)}} - 1$
Bổ đề 2: Nếu ${3^b}|{2^a} - 1 \Rightarrow {3^{b - 1}}|a$

Lời giải bài toán:
+ Khi $n=1$, bài toán thỏa mãn.
+ Khi $n>1 \Rightarrow n$ lẻ.
Gọi $p$ là ước nguyên tố lẻ nhỏ nhất của $n$ nên $\gcd \left( {p - 1,n} \right) = 1$.
Ta có: $p|{2^n} + 1|{2^{2n}} - 1$
Theo định lí Ferma nhỏ, ta có: $p|{2^{p - 1}} - 1$.
Áp dụng bổ đề 1, ta được: $p|\gcd \left( {{2^{p - 1}} - {{1,2}^{2n}} - 1} \right) = {2^{\gcd \left( {2n,p - 1} \right)}} - 1$
mà $\gcd \left( {2n,p - 1} \right) \leqslant 2 \Rightarrow p|3 \Rightarrow p = 3$
Đặt $n = {3^k}d$. Dùng bổ đề 2, ta có: ${3^{2k}}|{n^2}|{2^{2n}} - 1 \Rightarrow {3^{2k - 1}}|n \Rightarrow k = 1$
(*) Nếu $d>1$. Gọi $q$ là ước nguyên tố nhỏ nhất của $d$ nên $q \ge 5$.
Lập luận tương tự như trên, ta có: $q=7$
Vậy nên $7|n|{2^n} + 1$. Điều này vô lí vì ${2^n} + 1 \equiv 2,3,5(\bmod 7)$.
(*) Nếu $d=1$, ta có: $n=3$.
Vậy $n=1$ và $n=3$ thỏa mãn đề bài.



#312558 Bất đẳng thức $4$ biến thoả $ abcd=1$

Đã gửi bởi Stranger411 on 25-04-2012 - 08:17 trong Bất đẳng thức - Cực trị

Cho các số thực dương $a,b,c,d$ thoả mãn: $abcd=1$.Chứng minh rằng:
$$ 3(a^2+b^2+c^2+d^2) + 4 \ge (a+b+c+d)^2$$

Áp dụng bất đẳng thức Tukervici, ta có:
$ 3(a^4+b^4+c^4+d^4) + 4abcd \ge (a^2+b^2+c^2+d^2)^2$
Bất đẳng thức cần chứng minh là dạng tương đương của bất đẳng thức trên. $\blacksquare$
Bất đẳng thức trên dùng pp FMPX để chứng minh.



#346612 $\sum\limits_{k = 1}^{p - 1} {\l...

Đã gửi bởi Stranger411 on 13-08-2012 - 23:32 trong Số học

Cho p là số nguyên tố .CMR $\sum\limits_{k = 1}^{p - 1} {\left\lfloor {\frac{{{k^3}}}{p}} \right\rfloor } = \frac{{(p + 1)(p - 1)(p - 2)}}{4}$

Khuya rồi !! Lên Vmf chém thử vài bài cho vui :D

Với $1 \leqslant k \leqslant p - 1$thì $\left\{ \begin{gathered}
{k^3} \equiv 0(\bmod p) \\
{\left( {p - k} \right)^3} \equiv - {\left( k \right)^3} \\
\end{gathered} \right.$
Nên $\left( {\frac{{{k^3}}}{p} - \left\lfloor {\frac{{{k^3}}}{p}} \right\rfloor } \right) + \left( {\frac{{{{\left( {p - k} \right)}^3}}}{p} - \left\lfloor {\frac{{{{\left( {p - k} \right)}^3}}}{p}} \right\rfloor } \right) = 1$.
Từ đó, ta được:
$\begin{gathered}
\sum\limits_{k = 1}^{p - 1} {\left\lfloor {\frac{{{k^3}}}{p}} \right\rfloor } = \sum\limits_{k = 1}^{p - 1} {\frac{{{k^3}}}{p}} - \sum\limits_{k = 1}^{p - 1} {\left( {\frac{{{k^3}}}{p} - \left\lfloor {\frac{{{k^3}}}{p}} \right\rfloor } \right)} \\
= \frac{1}{p}\sum\limits_{k = 1}^{p - 1} {{k^3}} - \frac{1}{2}\sum\limits_{k = 1}^{p - 1} {\left( {\frac{{{k^3}}}{p} - \left\lfloor {\frac{{{k^3}}}{p}} \right\rfloor } \right)} + \left( {\frac{{{{\left( {p - k} \right)}^3}}}{p} - \left\lfloor {\frac{{{{\left( {p - k} \right)}^3}}}{p}} \right\rfloor } \right) \\
= \frac{1}{p}{\left( {\frac{{p\left( {p - 1} \right)}}{2}} \right)^2} - \frac{{p - 1}}{2} = \frac{{\left( {p + 1} \right)\left( {p - 1} \right)\left( {p - 2} \right)}}{4} \\
\end{gathered}$

ps Chú Nguyenta98 post trước mình 4 phút.